tzyc
Thanks Received: 0
Atticus Finch
Atticus Finch
 
Posts: 323
Joined: May 27th, 2012
 
 
trophy
Most Thankful
 

Q16 - Recent research shows that

by tzyc Wed Mar 20, 2013 7:08 am

Is this more like justify the conclusion question or assumption question? (I thought it is sufficient assumption question but when I saw the word "justify" I thought it's maybe justify q?)
And...basically we only see the last sentence starting from since and we can find the answer (D) right? The others do not play much roles I thought...
And, is (A) wrong because we are already told in the stimulus?
Thank you
Last edited by tzyc on Fri Mar 22, 2013 6:55 am, edited 1 time in total.
 
sumukh09
Thanks Received: 139
Atticus Finch
Atticus Finch
 
Posts: 327
Joined: June 03rd, 2012
 
This post thanked 2 times.
 
trophy
Most Thanked
trophy
First Responder
 

Re: Q16 - Recent research shows that

by sumukh09 Wed Mar 20, 2013 1:28 pm

This would be a sufficient assumption question. And yes, you're right, I would call A a premise booster as it just restates that the classical account was based on a faulty tenet (gradualness).

D is correct because it justifies the conclusion that the sound theory ought to be discarded. If all theories of sound change rely heavily on the classical theory then it must be discarded since recent research shows it to be confounded. In other words, because a theory was found to be based on something that was refuted (sound change not gradual), and that theory (classical theory) is relied heavily upon by a more general theory (sound theory), then that general theory must be discarded.
User avatar
 
tommywallach
Thanks Received: 468
Atticus Finch
Atticus Finch
 
Posts: 1041
Joined: August 11th, 2009
 
This post thanked 3 times.
 
 

Re: Q16 - Recent research shows that

by tommywallach Thu Mar 21, 2013 1:52 am

Hey Guys,

First off, there's no such thing as a "justify" question. This is a sufficient assumption, because the argument doesn't say the conclusion is necessary (necessary assumption questions are fairly easy to find, see questions 13 and 21 in this same section). Either way, we start by looking at the core:

Premise: Sound change isn't gradual. The classical theory of sound change depends on gradualness, so it must be discarded
Conclusion: Sound-change theory must be discarded.

The gap here should be obvious, just because one account of sound theory (the "classical" account) has to be thrown out doesn't mean the whole theory must be thron out.

(A) We already know that the central tent of the classical account is wrong, so knowing more reasons the classical account sucks doesn't do much for the argument.

(B) This is out of scope. Whether it's random or not, it ain't gradual.

(C) This is also out of scope. The reason why sound changes (culture clashes, for example) doesn't matter. All that matters is whether sound-change theory works.

(D) CORRECT! If all theories of sound change rely on the classical theory, then the fact that the classical theory sucks means that sound change theory in general must suck.

(E) This one is crazy out of scope.'

Hope that helps!

-t
Tommy Wallach
Manhattan LSAT Instructor
twallach@manhattanprep.com
Image
User avatar
 
WaltGrace1983
Thanks Received: 207
Atticus Finch
Atticus Finch
 
Posts: 837
Joined: March 30th, 2013
 
 
trophy
Most Thanked
trophy
Most Thankful
trophy
First Responder
 

Re: Q16 - Recent research shows that

by WaltGrace1983 Thu Jan 16, 2014 7:25 pm

At first I was thinking, "how in the world could this be a sufficient assumption answer choice?" I read a little bit quickly saw "rely heavily" as not completely reliant. The word "heavily" makes it seem like rely is less strong, as if we can rely heavily on something or lightly on something and one is more reliant than the other. I dunno. Either way, once I got rid of the word "heavily" it made much more sense!

I see it like this:

IF you have a theory of sound change THEN it must be dependent on the classical theory. In other words, NO classical theory means NO theory of sound change.

If we discard the classical theory, we must thereby discard the sound change theory in general. This makes our conclusion 100% awesome.

Everything else is fairly out of scope. I feel like this is par for the course in sufficient assumption?
User avatar
 
tommywallach
Thanks Received: 468
Atticus Finch
Atticus Finch
 
Posts: 1041
Joined: August 11th, 2009
 
 
 

Re: Q16 - Recent research shows that

by tommywallach Sun Jan 19, 2014 3:45 pm

Hey Walt,

I agree wholeheartedly!

-t
Tommy Wallach
Manhattan LSAT Instructor
twallach@manhattanprep.com
Image